Difference between revisions of "2025 AMC 8 Problems"

m (Warning/disclaimer)
m (Disclaimer)
Line 7: Line 7:
 
==Problem 2==
 
==Problem 2==
  
The 2025 AMC 8 is not held yet. Please do not post false problems.
+
The 2025 AMC 8 is not held yet. ‘’’Please do not post false problems.’’’

Revision as of 08:51, 18 February 2024

Problem 1

Let $m$ and $n$ be $2$ integers such that $m>n$. Suppose $m+n=20$, $m^2+n^2=328$, find $m^2-n^2$.

$\textbf{(A)}\ 280 \qquad \textbf{(B)}\ 292 \qquad \textbf{(C)}\ 300 \qquad \textbf{(D)}\ 320 \qquad \textbf{(E)}\ 340$

Problem 2

The 2025 AMC 8 is not held yet. ‘’’Please do not post false problems.’’’